Difference between revisions of "2022 AMC 10B Problems/Problem 15"

(Solution 3 (I didn't know Solution 1!)=)
(Solution 3 (I didn't know Solution 1!)=)
 
(One intermediate revision by the same user not shown)
Line 22: Line 22:
 
~numerophile
 
~numerophile
  
==Solution 3 (I didn't know Solution 1!)===
+
==Solution 3 (I didn't know Solution 1!)==
  
 
We have a slightly challenging problem :-(, but that's okay!
 
We have a slightly challenging problem :-(, but that's okay!
Line 44: Line 44:
 
===Remark 3.0.1===
 
===Remark 3.0.1===
  
Hi! It me again!
+
Hi! It me (Pinotation) again!
  
If you are confused about why
+
If you are confused about why \( a_1 \neq 3 \) or \( a_1 \neq 4 \) or something along those lines, we can prove by induction.
 +
 
 +
We have
 +
<cmath>
 +
S_n=\frac{n}{2}\big(2a_1+(n-1)\cdot 2\big)=n(a_1+n-1).
 +
</cmath>
 +
Then
 +
<cmath>
 +
S_{3n}=3n(a_1+3n-1),
 +
</cmath>
 +
so
 +
<cmath>
 +
\frac{S_{3n}}{S_n}=\frac{3(a_1+3n-1)}{a_1+n-1}.
 +
</cmath>
 +
 
 +
Base case \( n=1 \):
 +
<cmath>
 +
\frac{S_3}{S_1}=\frac{3(a_1+2)}{a_1}.
 +
</cmath>
 +
 
 +
Suppose for some \( n \) the ratio is independent of \( n \) and equal to a constant \( k \). Then
 +
<cmath>
 +
\frac{S_{3n}}{S_n}=\frac{3(a_1+3n-1)}{a_1+n-1}=k.
 +
</cmath>
 +
For \( n+1 \) we must also have
 +
<cmath>
 +
\frac{S_{3(n+1)}}{S_{n+1}}=\frac{3(a_1+3n+2)}{a_1+n}=k.
 +
</cmath>
 +
So
 +
<cmath>
 +
\frac{3(a_1+3n-1)}{a_1+n-1}=\frac{3(a_1+3n+2)}{a_1+n}.
 +
</cmath>
 +
Cross multiplying gives
 +
<cmath>
 +
(a_1+3n-1)(a_1+n)=(a_1+3n+2)(a_1+n-1).
 +
</cmath>
 +
Expanding,
 +
<cmath>
 +
a_1^2+4na_1+a_1- n -3 = a_1^2+4na_1+a_1+2n-2.
 +
</cmath>
 +
Simplifying,
 +
<cmath>
 +
- n -3 = 2n -2,
 +
</cmath>
 +
so
 +
<cmath>
 +
3n = -1.
 +
</cmath>
 +
This is impossible for integer \( n \). The only way the equality can hold for all \( n \) is if the proportionality factor between numerator and denominator is fixed. Comparing coefficients in
 +
<cmath>
 +
3(a_1+3n-1)=C(a_1+n-1),
 +
</cmath>
 +
we get \( C=9 \) and hence
 +
<cmath>
 +
3a_1-3=9a_1-9 \implies a_1=1.
 +
</cmath>
 +
 
 +
I hope this helped! If not, then you can check Solution 1, as it is practically the friendlier version LOL!
 +
 
 +
~Proof by Pinotation
  
 
==Video Solution (🚀 Solved in 5 min 🚀)==
 
==Video Solution (🚀 Solved in 5 min 🚀)==

Latest revision as of 01:02, 10 September 2025

Problem

Let $S_n$ be the sum of the first $n$ terms of an arithmetic sequence that has a common difference of $2$. The quotient $\frac{S_{3n}}{S_n}$ does not depend on $n$. What is $S_{20}$?

$\textbf{(A) } 340 \qquad \textbf{(B) } 360 \qquad \textbf{(C) } 380 \qquad \textbf{(D) } 400 \qquad \textbf{(E) } 420$

Solution 1

Let's say that our sequence is \[a, a+2, a+4, a+6, a+8, a+10, \ldots.\] Then, since the value of n doesn't matter in the quotient $\frac{S_{3n}}{S_n}$, we can say that \[\frac{S_{3}}{S_1} = \frac{S_{6}}{S_2}.\] Simplifying, we get $\frac{3a+6}{a}=\frac{6a+30}{2a+2}$, from which \[\frac{3a+6}{a}=\frac{3a+15}{a+1}.\] \[3a^2+9a+6=3a^2+15a\] \[6a=6\] Solving for $a$, we get that $a=1$.

Since the sum of the first $n$ odd numbers is $n^2$, $S_{20} = 20^2 = \boxed{\textbf{(D) } 400}$.

Solution 2 (Quick Insight)

Recall that the sum of the first $n$ odd numbers is $n^2$.

Since $\frac{S_{3n}}{S_{n}} = \frac{9n^2}{n^2} = 9$, we have $S_{20} = 20^2 = \boxed{\textbf{(D) } 400}$.

~numerophile

Solution 3 (I didn't know Solution 1!)

We have a slightly challenging problem :-(, but that's okay!

$\textbf{If you didn't come up with the Solution 1 intuition}$, then here is a more direct approach.

We want \(\frac{S_{3n}}{S_n}\) to be a natural (\(\frac{S_{3n}}{S_n} > 0\); basically a whole number \(\neq 0\)) number. Then only can the progression be incremental by 2.

Assume that \(a_1 = 1\). Then, \(a_2 = 3\), \(a_3 = 5\), etc. We take the first term \(n=1\), which is 1. Then \(3n\); the next 3 terms will sum to 9. \(\frac{9}{1} = 9\), and this is an

What about \(a_1 = 0\). We immediately see that this would be undefined, so we cannot have \(a_1 = 0\). So we say \(a_1 = 2\). Then the sum for \(n=1\) is 2, and for \(3n\); it is simply 6. This is \(6/2 = 3\), so it works!

Then, what about \(a_2 = 3\)? Then this means that the \(n=2\) sum is 4, and the \(3n\) sum is 36. This is \(36/4 = 9\), and this is the same as the difference before, proving that \(a_1 = 1\) is indeed correct.

And then? What about \(a_2 = 4\)? This means \(n=2\) sum is 6, and the \(3n\) sum is 42, but uh oh, the progression breaks! Therefore, the evens cannot work.

Therefore, \(a_1 = 1\), \(a_{20}\) is just the sum of the first 20 odd numbers, which is $\boxed{\textbf{(D) } 400}$

~Pinotation

Remark 3.0.1

Hi! It me (Pinotation) again!

If you are confused about why \( a_1 \neq 3 \) or \( a_1 \neq 4 \) or something along those lines, we can prove by induction.

We have \[S_n=\frac{n}{2}\big(2a_1+(n-1)\cdot 2\big)=n(a_1+n-1).\] Then \[S_{3n}=3n(a_1+3n-1),\] so \[\frac{S_{3n}}{S_n}=\frac{3(a_1+3n-1)}{a_1+n-1}.\]

Base case \( n=1 \): \[\frac{S_3}{S_1}=\frac{3(a_1+2)}{a_1}.\]

Suppose for some \( n \) the ratio is independent of \( n \) and equal to a constant \( k \). Then \[\frac{S_{3n}}{S_n}=\frac{3(a_1+3n-1)}{a_1+n-1}=k.\] For \( n+1 \) we must also have \[\frac{S_{3(n+1)}}{S_{n+1}}=\frac{3(a_1+3n+2)}{a_1+n}=k.\] So \[\frac{3(a_1+3n-1)}{a_1+n-1}=\frac{3(a_1+3n+2)}{a_1+n}.\] Cross multiplying gives \[(a_1+3n-1)(a_1+n)=(a_1+3n+2)(a_1+n-1).\] Expanding, \[a_1^2+4na_1+a_1- n -3 = a_1^2+4na_1+a_1+2n-2.\] Simplifying, \[- n -3 = 2n -2,\] so \[3n = -1.\] This is impossible for integer \( n \). The only way the equality can hold for all \( n \) is if the proportionality factor between numerator and denominator is fixed. Comparing coefficients in \[3(a_1+3n-1)=C(a_1+n-1),\] we get \( C=9 \) and hence \[3a_1-3=9a_1-9 \implies a_1=1.\]

I hope this helped! If not, then you can check Solution 1, as it is practically the friendlier version LOL!

~Proof by Pinotation

Video Solution (🚀 Solved in 5 min 🚀)

https://youtu.be/7ztNpblm2TY

~Education, the Study of Everything

Video Solution By SpreadTheMathLove

https://www.youtube.com/watch?v=zHJJyMlH9DA

Video Solution by Interstigation

https://youtu.be/qkyRBpQHbOA

Video Solution by TheBeautyofMath

https://youtu.be/Mi2AxPhnRno?t=1299

See Also

2022 AMC 10B (ProblemsAnswer KeyResources)
Preceded by
Problem 14
Followed by
Problem 16
1 2 3 4 5 6 7 8 9 10 11 12 13 14 15 16 17 18 19 20 21 22 23 24 25
All AMC 10 Problems and Solutions

These problems are copyrighted © by the Mathematical Association of America, as part of the American Mathematics Competitions. AMC Logo.png